¿Es el operador Momentum un postulado?

He estado estudiando los postulados de QM y viendo cómo derivar ideas importantes de ellos. Sin embargo, una cosa que no he podido deducir de ellos es la identidad del operador de cantidad de movimiento.

Para simplificar, solo estoy pensando en efectos no relativistas, giros, potenciales dependientes del tiempo y una dimensión espacial. También asumo que el operador de posición es simplemente una multiplicación por X , como en, estoy en el espacio de posición. Entonces el operador hamiltoniano es H = 2 2 metro 2 + V .

Sé que el operador de cantidad de movimiento es pags = i X .

Pero, ¿cómo llego allí desde los postulados? Sé que tiene sentido , ya que da como resultado el Teorema de Ehrenfest, la hipótesis de la longitud de onda de De Broglie, el Principio de Incertidumbre de Heisenberg (por X y pags ), siendo el operador de cantidad de movimiento el generador del operador de traslación, y posiblemente muchos otros teoremas deseables y correlaciones con la cantidad de movimiento clásica.

Pero ninguno de estos son postulados (al menos, no en los diversos formalismos que encontré), por lo que no puedes derivar pags = i X de ellos. Más bien, son consecuencias de ella. Necesitas conocer al operador de antemano para ver que son correctos. Sí, esto es solo semántica, pero ese es el problema central para mí:

Independientemente de cuánto sentido tenga, ¿es la identidad pags = i X (Bajo las suposiciones que hice) un Postulado, lo que significa que no puede derivarlo de otros postulados, ¿o de hecho puede obtenerse de ellos? Y en este último caso, ¿podría indicarme cómo?

Nota: Sé que hay muchos conjuntos de postulados diferentes y equivalentes para QM. Pero en ninguno de los que vi lo nombraron como postulado ni lo derivaron propiamente.

Respuestas (7)

Primero, es un poco difícil escribir algo como:

PAGS ^ = i / X .
Es más riguroso escribir:
X | PAGS ^ | ϕ = i X X | ϕ ,
y debe interpretarse como el operador de impulso en la representación espacial.

Derivaciones:

El significado físico detrás del impulso es que: 1. Es la cantidad conservada correspondiente a la simetría de traslación espacial. 2. Debido a 1, el operador de cantidad de movimiento (hermitiano) es el generador del operador de traducción espacial (unitario).

En términos de ecuaciones:

Definir el operador de traducción espacial D ( a ) S t

C | X + a = D ( a ) | X ,
y:
D ( a ) = mi i a pags ^ /

Supongo que no tienes problemas para derivar esto.

Tenga en cuenta que esto solo depende de la condición de cuantificación [ X , pags ] = i , que es uno de los postulados de la mecánica cuántica.

Tomar un estado arbitrario | ϕ y aplicar D ( a ) en eso:

D ( a ) | ϕ = D ( a ) | ϕ | X X | d X
Cambio de variable, RHS =
C | X X a | ϕ d X

Tomar a 0 , conéctelo a RHS:

ϕ ( X a ) = ϕ ( X ) a X ϕ ( X )
y a LHS:
D ( a ) = 1 i a pags ^ /

puedes recuperar X | PAGS ^ | ϕ = i X X | ϕ

Por supuesto que tiene razón acerca de que la expresión de un operador depende de la representación elegida, pero cuando presento esta idea es para los estudiantes que tienen su primer encuentro con QM. Por el momento, el SE en una representación de posición es para ellos mecánica cuántica. Por lo demás, no veo una distinción significativa entre decir que voy a asumir "la forma del operador de cantidad de movimiento es esta " y decir "el conmutador [ X , pags ] es eso ".
¡Gracias! Entonces, la respuesta a mi pregunta sería algo como esto: Primero, defina el impulso en QM. La definición es 1., y de ella obtienes 2., que es, "el operador P genera el operador de traducción". Segundo, use un Postulado, [x,p] = iℏ, para mostrar que la condición anterior exige que ⟨x|P^|ϕ⟩=−iℏ∂⟨x|ϕ⟩/∂x. Entonces, al final, HAY un postulado con respecto a la forma de p.
@dmckee tienes razón!
Lo siento @HanaKaze, pero no creo que sea lógico decir "Ahora déjame deducirlo por ti" y luego hacer una serie de suposiciones sobre el marco de la mecánica cuántica. La mecánica clásica no dice [ X , pags ] = i , y de hecho mencionaste esto en tu respuesta, por lo que nada se "deriva". Todas estas son motivaciones , y hermosas por decir lo menos. Lea mi respuesta a continuación.
No puedo evitar pensar que la parte "Primero, es totalmente incorrecto escribir algo como PAGS = i / X ” está totalmente equivocado en sí mismo. Es común trabajar en el espacio de Hilbert. H = L 2 ( R ) de funciones de onda en la representación x. No hay nada malo en ello. Por el contrario, se podría argumentar que confiar en los "mercados propios de posición | X ” para definir algo está mal, ya que el operador de posición como operador autoadjunto en H no tiene vectores propios.
@ m93a Estoy de acuerdo contigo en que no debería decir que algo está totalmente mal. Tengo plena fe en que los físicos entienden lo que eso significa, pero soy demasiado perezoso para escribirlo con rigor. En segundo lugar, su comentario 'se podría argumentar que depender de los "elementos propios de posición ∣∣𝑥⟩" para definir algo está mal' es exactamente lo que traté de demostrar en mi respuesta, por esta razón, escrito vagamente P = −𝑖ℏ∂/∂ 𝑥 no es una buena práctica, incluso si sabes que así es como se ve en la representación espacial.
sabes que el comentario anterior fue escrito hace 5 años?
@ziv I tbh vi ese comentario de mi notificación y lo respondí... ni siquiera me di cuenta de su marca de tiempo...
@HanaKaze Tu publicación está llena de cosas matemáticamente mal definidas. Nunca es “más riguroso” escribir algo con X | , como usted dice. El operador i d d X en L 2 ( R ) es perfectamente riguroso ( fuente ) y es utilizado todo el tiempo por los físicos matemáticos. La expresion d d X X | ϕ no está definido a menos que especifique una manipulación del espacio de Hilbert ( fuente ). Tu publicación es simplemente incorrecta.
Está utilizando la palabra "riguroso" sobre QM sin comprender realmente la formulación rigurosa de QM. Por favor, no hagas eso, desorienta a otras personas. Reconoce la inexistencia de vectores propios del operador de posición, pero usa | X en expresiones formales como si existieran. Esto no es rigor. Ni por asomo. No hay nada malo a priori en las derivaciones formales, pero, por el amor de Dios, no las llames rigurosas . Y no llame a todos los que hacen QM de manera diferente a usted como incorrectos (o rudimentarios ).
@m93a gracias por tus comentarios, eres exactamente la razón por la que dejé la academia. Siéntase libre de rechazar la respuesta o eliminarla o editarla.
Querida @HanaKaze, lamento mucho haberte hecho sentir mal, debería haber redactado mis comentarios de otra manera. No tengo los privilegios para editar las publicaciones de otros, propuse una edición que mantuvo el núcleo de su derivación pero cambió la redacción para que sea mucho más precisa. La edición fue rechazada por mods, no sé por qué, no ha habido discusión. Por lo tanto, el único lugar donde puedo corregir o discutir su publicación es la sección de comentarios.

No hay derivación, pero hay un argumento heurístico.

Supongamos que estamos en 1926 y Derby nos acaba de desafiar a mostrarle la ecuación de onda que va con las "ondas" de De Broglie (como desafió a Schrödinger). Eso significa que estamos trabajando en una ecuación de onda. Las soluciones deben ser de la forma (en una dimensión)

Ψ ( X , t ) = A mi i ( k X ω t )
dónde k = 2 π / λ es el número de onda y ω = 2 π / T es la frecuencia angular.

también queremos

pags = h / λ = k mi = h F = ω
estar de acuerdo con las suposiciones ad hoc de de Broglie y Plank que están funcionando .

Podríamos notar (como supongo que hizo Schrödinger) que las derivadas espaciales y temporales que suelen aparecer en una ecuación de onda nos darán factores de k y ω respectivamente (con algunos factores inconvenientes de i dando vueltas, pero tenemos que vivir con eso). Es decir, acabamos de decidir ir con

pags i X mi i t

A partir de ahí, solo es cuestión de decir que para una partícula que se mueve en un potencial V la energía total (Hamiltoniana en muchos casos) es

mi = T + V = pags 2 2 metro + V ,
Al ver esto como una derivada con respecto al tiempo y dos derivadas con respecto al espacio y luego arreglar las constantes, podemos llegar a
(TDSE) [ 2 2 metro 2 X 2 + V ( X ) ] Ψ ( X , t ) = i t Ψ ( X , t )

Debo reiterar que esto no es de ninguna manera una prueba. Es una especie de argumento de plausibilidad extendida. Y uno que más bien tensa la suspensión de la incredulidad excepto que funciona.


Tengo una versión más cuidadosamente construida de este argumento que doy a mis estudiantes de física moderna y se pueden encontrar variaciones en muchos lugares que son anteriores a mi versión.

Hay una derivación, utilizando el teorema de Noether y el límite cuasi-clásico, cf. Landau QM sec. 15: libros.google.co.uk/…
@bolbteppa La cuestión es que el límite cuasiclásico es un procedimiento justificado porque funciona (es decir, un postulado) en lugar de algo que surge inexorablemente de las matemáticas. Tenemos que fundar QM en algún conjunto de suposiciones. El OP pregunta si el significado de pags ^ en la representación de posición es uno de ellos y dices "no, puedo usar una suposición diferente". Lo cual es cierto pero elude el problema porque las dos suposiciones son equivalentes entre sí.
Como dije antes, entiendo por qué la forma de p tiene sentido. Pero al final parece que hay que darlo como postulado, directa o indirectamente. Si algo no se puede derivar, se tiene que postular. Así es como yo veo las cosas, al menos.

El hecho de que PAGS ^ i / X en la posición la base no es ni derivable ni postulado, porque no siempre es cierto. La relación de conmutación canónica [ X ^ , PAGS ^ ] = i yo ^ generalmente se toma como un postulado, pero incluso si elige la representación X ^ X en la base de posición, entonces el CCR permite infinitas representaciones de PAGS ^ de la forma PAGS ^ ( i / X ) + F ( X ) para cualquier función F ( X ) . La elección de la representación corresponde a una elección de calibre para la función de onda y no afecta a ninguna cantidad observable físicamente. Consulte el Ejercicio 7.4.9 en las págs. 213-214 de Shankar para mayor discusión.

Sé que el operador de cantidad de movimiento es P = -iℏ ∂/∂x.

Sin duda, es el operador de cantidad de movimiento en la base de posición . El operador de cantidad de movimiento en la base de cantidad de movimiento es PAGS = pags en analogía con el operador de posición en la base de posición es X = X .

(Tomado prestado en gran medida de la "Teoría del campo cuántico de partículas puntuales y cuerdas" de Brian Hatfield)

La clave es comenzar con la relación de conmutación.

[ X , PAGS ] = i

Si | X denota un estado propio de posición, entonces

X | X = X | X

y

X | X | X = X d ( X X )

lo que quiere decir que el operador X es diagonal en la base de la posición. Nosotros buscamos

X | PAGS | X

Ya que

[ X , X ] = 1

se sigue que el operador i X sirve como representación de PAGS en esta base y por lo tanto

X | PAGS | X = i X d ( X X )


Si el operador hamiltoniano es

H = PAGS 2 2 metro + V ( X )

después

X | H | X = ( 2 2 metro 2 X 2 + V ( X ) ) d ( X X )

Ahora, la ecuación de Schrödinger es

i t X | ψ ( t ) = X | H | ψ ( t )

Insertar la identidad

1 = d X | X X |

rendimientos

i t X | ψ ( t ) = i t ψ ( X , t ) = d X X | H | X X | ψ ( t ) = d X X | H | X ψ ( X , t )

y finalmente, usando el resultado de la parte superior de esta sección,

i t ψ ( X , t ) = d X ( 2 2 metro 2 X 2 + V ( X ) ) d ( X X ) ψ ( X , t ) = ( 2 2 metro 2 X 2 + V ( X ) ) ψ ( X , t )

Tienes el álgebra abstracta:

[ X , pags ] = i

como postulado. O bien proviene del corchete de Poisson habitual que va a la regla del conmutador, o simplemente se establece de manera abstracta como una definición.

De todos modos, puedes buscar representaciones de este álgebra. Lo primero que hay que ver es que no hay representaciones de dimensión finita (también conocidas como matrices). Una prueba por absurdo pasa por suponer que es posible y luego se debe tomar la traza de la relación de conmutación. Es obvio que obtienes

1 = 0 .

Lo segundo que deberías ver: esta es una traducción infinitesimal. De hecho, considere a un parámetro infinitesimal, entonces

d X = [ X , a pags ] = i a ,

que es la traducción infinitesimal habitual que cabría esperar. Usted es naturalmente llevado a pensar en esto. a pags como generador de traducciones.

Desafortunadamente, la clasificación de representaciones de álgebras de dimensión infinita es un tema sutil. Te señalo el teorema de Stone-von Neumann .

Lo mejor que puedo hacer es motivar a la representación habitual. Y en realidad no es tan difícil, porque solo nos queda el álgebra de difeomorfismo (recuerde que debe ser de dimensión infinita), donde X y pags debe actuar sobre las funciones.

Dada una función de x, llamada ψ ( X ) , se puede obtener una traslación por la serie de Taylor:

ψ ( X + a ) = ψ ( X ) + a ψ ( X ) + a 2 2 ψ ( X ) + = Exp ( a d d X ) ψ ( X ) ,

Y ahí lo tienes: pags = i d d X . Luego, el álgebra se realiza mediante el álgebra de campo vectorial habitual :

L pags X = i .

Verá, los derivados siempre generan traducciones. La Mecánica Cuántica te dice que los llames impulso.

Les dejo a ustedes que averigüen qué hubiera pasado si hubiera optado por actuar en funciones de pags .

Otra respuesta que sugiere que hay un postulado específicamente para la forma de p (con respecto a x). Gracias. Supongo que los libros de texto que leí no eran tan completos.
@JuanPerez No, ¿por qué dices eso? Es una representación. El contenido de la teoría son estados y operarios donde la norma da probabilidad. Una representación es solo una forma de representar estas cosas. ¡No cambia el contenido físico! Tampoco es necesario, es simplemente conveniente.
Acabas de decir que tomas el álgebra abstracta [x,p] = iℏ como postulado. Lo que quise decir es que ese postulado da específicamente la relación entre x y p, y que no podrías obtener una expresión funcional del operador p (que, como dices, depende de la representación que elijas) sin él.
Desafortunadamente, no creo que me haya dejado claro. No hay necesidad de un postulado adicional para obtener P = d/dx. Cualquier cosa que satisfaga [x,P]=i es válida. Y puede escribir estas representaciones de la misma manera que obtiene las Matrices de Pauli solo del álgebra de momento angular. Todas las respuestas de estas personas te muestran exactamente lo mismo.
Siento seguir respondiendo, pero estoy un poco confundido. Ahora entiendo que P = -iℏd/dx no es un postulado. Pero [x,p] = iℏ es uno, ¿verdad? (Si no es un postulado, sino una elección, podría elegir otra cosa, como [x,p] = 0, o [x,p] = 78, ¿y obtener resultados equivalentes?)
Respuesta de dos piezas. 1) Por favor, no digas que lo sientes. Se anima a hacer preguntas. Acerca de tu pregunta: Bueno, tienes que empezar desde algún lado. Ahora te das cuenta de que lo importante es [x,p] = número distinto de cero independiente de p y x. Cualquier número está bien, porque puedo absorberlo y redefinir x y p para obtener las cosas habituales (nadie te dijo que deberían corresponder directamente a las cosas clásicas). La parte no trivial es motivar una relación. [ X , pags ] 0 .
2) Ahora debe comenzar desde algún lugar para obtener esto. A la gente de aquí parece gustarle los argumentos de simetría. Pero podrías haber partido del principio de incertidumbre, por ejemplo. Cualquier cosa física que se te ocurra que te lleve a pensar en este álgebra será suficiente.
Sí. Tienes que empezar desde algún lado . Cualquiera de esos puede ser el postulado relevante, pero uno de ellos tiene que serlo. No puedes simplemente deducirlos de los otros postulados.

Como una extensión rápida de las respuestas anteriores, permítanme repetir que nada de la mecánica cuántica se "deriva" de ninguna teoría anterior. Sí, hay muchas correspondencias que son bastante llamativas: cuantización canónica, cuantización geométrica, ondas de acción en la teoría de Hamilton-Jacobi, extensión de la relación de dispersión de DeBroglie (de lo que hablaba @dmckee), etc., y muchas personas las utilizan para motivar a los desarrollo de la mecánica cuántica desde el punto de vista de la física clásica. Pero al final del día, la mecánica cuántica es la teoría más fundamental, por lo que se postula (los llaman "postulados de QM" por una razón :).

Otra forma de verlo es que recuperar la mecánica cuántica de la física clásica no es un problema bien planteado. La información se pierde cuando se toma el "límite clásico" de la mecánica cuántica, por lo que "derivar" la mecánica cuántica de la física clásica en su definición estricta no tiene sentido.

Este mensaje es moralmente idéntico al que destaca Feynman en este popular vídeo .

Se afirma que "nada de la mecánica cuántica se 'deriva' de ninguna teoría anterior". Esto no es correcto. La mecánica cuántica se basa en la teoría clásica de campos.
¿Podrías definir "fundado"?

La forma "fundamental" más adecuada es derivarlo del conmutador , que básicamente te dice cómo funciona el intercambio de información entre la posición y el momento, que es el corazón de la lección de la mecánica cuántica: el Universo contiene un límite de contenido de información, al igual que tiene un límite de velocidad de información. Tenga en cuenta que el operador de impulso solo se ve así con respecto a la posición , por lo que, en cierto sentido, esto también supone que también hemos definido la posición.

Es un hecho empírico, establecido a niveles obscenos de confianza, por muchas pruebas y fracasos repetidos, que es imposible consultar más información de un sistema con respecto a su posición y momento juntos que la dada por el límite

H X + H pags 1 + lg ( π )

expresada a través de la entropía de Shannon (tenga en cuenta que el límite depende de las unidades que esté utilizando; técnicamente, la entropía es relativa a una escala), o más crudamente (no tan fuerte, es decir, hay casos en los que la relación inferior se mantiene pero no la superior). , y no son casos físicamente válidos, por ejemplo, la suma de dos funciones delta adecuadamente separadas tanto en el espacio posicional como en el momentáneo) y normalmente se dan como

Δ X Δ pags 1 2

.

En el lenguaje basado en álgebra lineal que proporciona la teoría cuántica, eso significa que los operadores X ^ y pags ^ debe satisfacer

[ X ^ , pags ^ ] = i

Si tiene dos operadores cualesquiera que satisfagan esto, entonces es posible demostrar que si usa los estados propios de uno de ellos como base , indexados por X abajo, entonces el otro debe tener la forma

pags ^ = i X

. Así que ni siquiera necesita tratar de averiguar cuál es la base para X ^ o pags ^ es, simplemente asuma que existe, y derive en consecuencia. Si usó una base diferente a la que usó otra persona, las matemáticas seguirán funcionando de la misma manera.

Por lo tanto, el operador de cantidad de movimiento en sí mismo no sería un postulado directamente. Más bien, como parte de la descripción de una sola partícula, deberíamos especificar la relación de conmutación entre la posición y el momento, y eso los define a ambos a la vez.